Showing a set of matrices is a direct sum.

Click For Summary
The discussion focuses on proving that the space of n x n matrices, MnXn, is the direct sum of the symmetric matrices W1 and the skew-symmetric matrices W2. The definition of direct sum requires showing that every matrix in MnXn can be expressed as a sum of matrices from W1 and W2, and that their intersection is only the zero matrix. Participants clarify the misunderstanding of using the same matrix for both W1 and W2, emphasizing that distinct matrices must be used. The manipulation of matrices is explored, with attempts to express the sum of matrices from W1 and W2 correctly. The conversation highlights the importance of correctly applying the definitions of symmetric and skew-symmetric matrices in the proof.
trap101
Messages
339
Reaction score
0
Let W1 = {A\in MnXn(R)| A = AT} and W2 = {A\in MnXn(R)| A = -AT}

Show that MnXn = W1 (+) W2

where the definition of direct sum is:

V is the direct sum of W1 and W2 in symbols:

V = W1 (+) W2 if:

V = W1 + W2 and
W1 \cap W2 = {0}


Attempt:

I figure I have to show each property individually. So for the first property I tried to do a manipulation:

AT + (-AT) = (A + (-A))T = 0T

Then I added A to each side: A + (A + (-A))T = A + 0T

Did I even show what was needed?
 
Physics news on Phys.org
No, you appear to be misunderstanding what "W1= {A\in Mxn|A= AT}" and "W2= {A\in Mnxn|A= -AT[/sub]}" mean. Members of V are of the form A+ B where A is in W1 and B is in W2. You cannot use the same matrix, A, for each.
 
HallsofIvy said:
No, you appear to be misunderstanding what "W1= {A\in Mxn|A= AT}" and "W2= {A\in Mnxn|A= -AT[/sub]}" mean. Members of V are of the form A+ B where A is in W1 and B is in W2. You cannot use the same matrix, A, for each.



How's this:

Letting A be a matrix from W1 and B be a matrix from W2:

V = A + B
= AT+ (-BT)
= (A+(-B))T

Can I bring out the -1 and have: (-1)(A+B)T?
 
Question: A clock's minute hand has length 4 and its hour hand has length 3. What is the distance between the tips at the moment when it is increasing most rapidly?(Putnam Exam Question) Answer: Making assumption that both the hands moves at constant angular velocities, the answer is ## \sqrt{7} .## But don't you think this assumption is somewhat doubtful and wrong?

Similar threads

Replies
4
Views
2K
  • · Replies 4 ·
Replies
4
Views
5K
  • · Replies 2 ·
Replies
2
Views
3K
  • · Replies 3 ·
Replies
3
Views
6K
  • · Replies 2 ·
Replies
2
Views
2K
Replies
7
Views
4K
Replies
5
Views
8K
  • · Replies 2 ·
Replies
2
Views
4K
  • · Replies 15 ·
Replies
15
Views
2K
  • · Replies 2 ·
Replies
2
Views
3K